0 Daumen
464 Aufrufe

 

Könnt ihr mal rüberschauen? 

Die richtige Lösung müsste t= 1,4 sein

-50 e^ -0,5t +100 e^-t = 0    |+50 e^-0,5t

100 e^-t = 50 e^-0,5t           | :50 

2 e^-t = e^-0,5t                | ln

-2t = -0.5t 

4t = t


Irgendwas maxge ich falsch. :/

Avatar von

Stimmt denn die Gleichung ?

Jap, hab eben sogar nachgeschaut - die stimmt.

1 Antwort

0 Daumen
Links sollte nach dem Logarithmieren \(\ln(2)-t\) stehen.
Avatar von

Was muss ich danach machrn?

Ln (2) - t = -0.5t 

0.69 -t = -0.5t     | +t 

0.69 = -0.5 t^2      | :(-0.5) 

T^2 = -1,38           Wurzel 

Schon wieder falsch. Was ist jetzt wieder falsch?

Ein anderes Problem?

Stell deine Frage

Willkommen bei der Mathelounge! Stell deine Frage einfach und kostenlos

x
Made by a lovely community